Assignment 4

Exercise 1 Give an example of the following, or explain why no such example can exist.

  • A sequence with infinitely many terms equal to zero, but does not converge to zero.
  • A sequence with infinitely many terms equal to zero, which converges to a nonzero number.
  • A sequence of irrational numbers that converges to a rational number.
  • A convergent sequence where every term is an integer.
  • Two divergent sequences an,bn where an+bn is convergent.
  • Two divergent sequences an,bn where anbn is convergent.

Solution.

  • This is possible: the sequence 0,1,0,1,0,1,0,1, has every other term equal to zero but is divergent.

  • This is impossible. If sna and a0, then for ϵ=|a|/2 we can find an N for which all n>N have |sna|<ϵ. Unpacking this, a|a|/2sna+|a|/2 Whether a is greater than or less than zero, this interval does not contain zero. So, no terms past N can possibly be zero, and there must only be finitely many terms equal to zero (they can only be in the terms s1,s2,,sN)

  • This is possible: given any irrational x the sequence x/n converges to 0 by the limit theorems.

  • This is possible: take the constant sequence 1,1,1,1,1,1,

  • Let an=n and bn=n. Then each sequence diverges, but an+bn=n+(n)=0 is the constant sequence 0,0,0, which converges to 0.

  • Let an=0,1,0,1,0,1,0, and bn=1,0,1,0,1,0,1,. Each of these is divergent but their product anbn is the zero sequence 0,0,0,0 so this is convergent.

Exercise 2 Prove, directly from the definition of convergence (not using limit theorems / squeeze theorem / etc), that 2n25n+125

Solution. Let ϵ>0. Choose N=125ϵ25ϵ. Since ϵ0, this fraction is defined. Now, let n>N. Then we have 125ϵ25ϵ<n1225ϵ5<n12ϵ5<25n12ϵ<25n+5 Since ϵ>0, we can multiply both sides of this inequality by ϵ. 12<(25n+5)ϵ1225n+5<ϵ Since nN, we have 1225n+5>0, so |1225n+5|=1225n+5. By previous proof, we have |x|=|x|. Therefore, we can rewrite our inequality as |1225n+5|<ϵ|10n1010n25(5n+1)|<ϵ|10n105(5n+1)10n+25(5n+1)|<ϵ|2n25n+125|<ϵ Therefore, for any arbitrary ϵ, there exists an N such that for all n>N, |2n25n+125|<ϵ. Thus, by definition, 2n25n+125.

Exercise 3 If an is a convergent anL for all n, then limanL. Similarly prove if an is a convergent anU for all n, then limanU.

To gether these give a very useful way to bound a limit: if sn is a convergent sequence with LsnU for all n, then LlimsnU.

Solution. Let ana and assume for contradiction a<L. Then L=a+β for some β>0. Set ϵ=β2. The interval (aϵ,a+ϵ) contains numbers strictly less then L. By the definition of convergence, we know there exists an N s.t. for all n>N, an lies in this interval. But this is a contradiction, because we know anL for all n, so we know aL.

A similar argument works for the other part. Let anb. For the sake of contradiction, suppose a>U. Then U=bβ for some β>0. Then, for some ϵ>0, we can let ϵ=β2. Thus The interval (aϵ,a+ϵ) contains numbers strictly greater then U. By the definition of convergence, we know there exists an N s.t. for all n>N, an lies in this interval. But this is a contradiction, because we know anU for all n, so we know aU.

Another way of expressing this idea:

Solution. First we consider the lower bound: assume for the sake of contradiction that ana but a<L even though for all n we know anL. Set ϵ=La2: then we can find an N where for all n>N we know |ana|<ϵ.
Unpacking this shows La2<ana<La2 which implies (looking only at the upper bound, as that’s what’s relevant for us) an<a+La2=L+a2 Thus, an must be less than the average of a and L, but since a<L this average is less than L! This is a contradiction, since we know anL for all n.

A very similar proof works for the upper bound: if ana and a>U even though anU for all n, setting ϵ=aU2 allows us to get an N where |ana|<ϵ, whose lower bound unpacks into aaU2=a+U2<an

Thus, an must be greater than the average of a and U, which (since a is greater than U itself) must be greater than U, contradicting the fact that anU.

Exercise 4 Let xn,yn be two convergent sequences with the same limit. Prove directly (ie don’t just use the squeeze theorem) that the interleaved sequence sn defined by x0,y0,x1,y1,x2,y2,x3,y3,,xn,ym also converges, and has the same limit.

Solution. Call the common limit of xn and yn by L. Choosing an ϵ>0 we know that we can find an Nx such that |xnL|<ϵ for all n>Nx, and an Ny such that $|y_n-L|<for all n>Ny. Now let N=4max{Nx,Ny} (the factor 4 is overkill and you can do better, but I’m not going to bother).

Choosing an arbitrary n>N, we want to show that |snL|<ϵ. Since sn is made from a mix of terms from xn and yn we have two cases. First consider if n is even, so we can write n=2k, and sn=yk as the terms are interleaved every other. Now, as n>N we see n>4maxNx,Ny implies that k>2maxNx,Ny and so in particular, k>Ny. Thus, by our assumption on Ny, we have that |ykL|<ϵ, so |snL|<ϵ as desired.

The reasoning is identical for n odd, where now n=2k+1 and sn=xk. Since n>4max{Nx,Ny} we know n1=2k>3max{Nx,Ny}, thus k>maxNx,Ny so |xkL|<ϵ or equivalently |snL|<ϵ.

Exercise 5 Use the squeeze theorem to prove that lim(n321n33n3+5)2n+7=0 Hint: show this sequence is positive for all large enough n, and then bound it above by an easy-to-compute sequence that we can show goes to zero. Notice how much easier this is than trying to find an N for a given ϵ!

Solution. Starting with the given fraction, two things that we can do to make a larger sequence it is to make it a smaller denominator, and a larger numerator, (n321n33n3+5)2n+7<(n33n3)2n+7(n321n33n3+5)2n+7<(13)2n+7 And a way to make a smaller sequence is to use the sequence 0. This is because, after n>1 the fraction stays positive.

0(n321n33n3+5)2n+7(13)2n+7

Now we just need to work with the upper bound a little more: since 2n+7>n we see that 32n+7>3n and

(13)2n+7<(13)n

That means we can replace our upper bound with this:

0(n321n33n3+5)2n+7<(13)2n+7<(13)n

And since we have proven that an0 and if |a|<1 we can apply this to (1/3)n and see it goes to zero.

0lim(n321n33n3+5)2n+7(13)n=0

So we can see that the limit must be 0, via the squeeze theorem.

Exercise 6 Prove that for every xR there exists a sequence yn of irrationals with ynx.

Solution. Let xR be arbitrary. We know that the sequence an=x has the property that anx. Similarly, we know that the sequence bn=1n has the property that bn0. So, by the sum law for limits, we know that lim(an+bn) = liman + limbn = x + 0 = x. Let’s define the sequence cn as cn=1n+x, so cnx.

This means that there exists a sequence cn such that cnx, but note that this doesn’t require that cn is solely irrational numbers. However, by the density of the irrationals in the real numbers (proved in an earlier homework), we know that there exists some irrational number in between any m,nR. So, lets set m=cn and n=x. According to the density of the irrationals, there must be an irrational number y such that xycn for all nN. Lets form a sequence yn, where yn=y, dictated by the inequality in the previous sentence. This sequence is made of only irrational numbers, and satisfies the equation anyncn, so by the squeeze theorem, ynx.

Exercise 7 In this problem you give an alternative proof that a1/n1 for all a>0 than the textbook’s (which used Bernoulli’s inequality). Instead, you’ll make use of the geometric sum from two assignments ago!
To recall, this stated that for any |r|<1 and nN, 1+r+r2+rn1=1rn1r

  • First consider only the case that a>1. Show that the geometric sum can be rewritten rn1=(r1)(1+r+r2++rn1), and use this to prove that a1n(a1/n1). Hint: apply it to r=a1/n and do some estimating.
  • Use this to show that 0a1/n1a1n for all n, and then prove (either directly, or using the squeeze theorem) that this implies a1/n1.
  • Now consider the case 0<a<1 and show the same. Hint: if 0<a<1 then 1/a>1, and perhaps you can do a similar trick to the textbook?

Solution. We begin with algebra: clearing denominators and multiplying the geometric sum from a previous homework by 1 to get in the desired form. Substituting in r=a1/n to the geometric sum equality rn1=(r1)(1+r+r2++rn1) gives a1=(a1/n1)(1+a1n+a2n++an1n) In the case a>1, we know by previous homework that taking nth roots and powers preserves inequalities, so a1/n>11/n=1 and so ap/n>1p=1. Thus, 1+a1/n+a2/n++a(n1)/n>1+1++1=n as there are n terms in the sum. This gives the inequality we were after: a1=(a1/n1)(1++a1/n+a2/n++a(n1)/n)>(a1/n1)n

Dividing this inequality by n gives an upper bound for a1/n1: a1/n1<a1n. And recalling that we already know a1/n>1 we see the left side of this inequality is positive so we’ve squeezed a1/n1 between two known quantities: 0<a1/n1<a1n

From here we prove directly that a1/n1, though you may also choose to combine the squeeze theorem and limit laws for sums. Let ϵ>0 and choose N=a1ϵ. Then for n>N we are guaranteed 0<a1/n1<a1n<a1N=a1a1ϵ=ϵ

and so |a1/n1|<ϵ, meaning lima1/n=1.

Next, we consider the case where a<1. This means b=1/a is greater than 1 and so the above argument applies here: lim(b)1/n=1

Using our limit laws, we can find the limit of the reciprocal (since b1/n0 and the limit is nonzero)

lim1b1/n=1limb1/n=11=1

But 1/b=1/(1/a)=a! Thus

lima1/n=lim(1b1/n)1/n=lim1b1/n=1

Exercise 8 (Limit of Products and Reciprocals) The textbook gives the scratch work of an argument for proving the following two claims:

  • If sn,tn are convergent sequences then sntn converges, and lim(sntn)=(limsn)(limtn)
  • If sn is a convergent sequence where sn0 and limsn0, then 1/sn is convergent, with lim1sn=1limsn.

Take the scratch work for these two theorems and rewrite as rigorous proofs (starting by choosing an ϵ, then proposing and N in terms of ϵ, and finally proving that the required inequality holds for all n>N.)

Solution. This is just asking you to write rigorously (and in the right order) the sketch argument provided in the textbook, for practice.

Exercise 9 Use the limit laws to rigorously prove the following sequence converges, and to calculate its limit.

Hint: to apply the limit laws we need to work from the “inside out”: that is, before concluding the limit of a sum exists we need to first prove the limit of each of the terms does! Hint 2: think about the kind of “algebra simplifying” you do in Calc 1! Do this same algebra, to then rigorously apply limit laws together with the fact that lim1/n=0 sn=12n+n213n2n+1

Solution. We start with the inside most fraction n213n2n+1. Dividing the numerator and denominator both by n2 yields the equivalent fraction 11n231n+1n2 To prove the numerator converges, we use the limit theorems for products and sums. Since 1/n0 we know 1/n2=1/n1/n converges with limit 00=0. Subtracting from the constant sequence 1, we see lim11n2=lim1lim1n2=10=1. The denominator is similar: since 1/n20 we see lim(31n+1n2)=lim3lim1n+lim1nlim1n=30+00=3 Since additionally the denominator is never zero (as 3n23n so 3n2n2n0 and 3n2n+11), the limit law for quotients applies to give limn213n2n+1=lim11n231n+1n2=lim(11n2)lim(31n+1n2)=13

Now, as each term in this sequence of fractions is positive, we can apply the limit law for square roots of convergent sequences to conclude limn213n2n+1=13

Since 12n=(12)n we can apply our basic limit an0 for |a|<1$ to see 12n0, and thus by the limit law for sums

lim{12n+n213n2n+1}=lim12n+limn213n2n+1=0+13=13

Finally, we make one more applicaiton of the limit theorem for roots: since the limit just calculated is nonnegative, we have

lim12n+n213n2n+1=13=134